Search found 160 matches


I would go with C

Can we have the OA?

by sanp_l

Sat Sep 25, 2010 10:29 pm
Forum: Critical Reasoning
Topic: CR - The Department of Homeland Security
Replies: 11
Views: 5119

@ptm_30: I feel there is a difference between "outside information" and a fact of common knowledge. That in a democracy the people are expected to participate in the nation's political decision making is already stated. That US has a democracy is of common knowledge. "Democracy" ...

by sanp_l

Thu Sep 23, 2010 12:44 pm
Forum: Critical Reasoning
Topic: Christopher Columbus Analogy
Replies: 11
Views: 7635

Hi, My approach lead me to E which is as below: The analogy is between Columbus falsifying the log deliberately to curb a sailor's mutiny and discover the main world and misleading information by the U.S. government in support of its policies and programs. Both aren't the same as the former involves...

by sanp_l

Wed Sep 22, 2010 10:15 pm
Forum: Critical Reasoning
Topic: Christopher Columbus Analogy
Replies: 11
Views: 7635

Hi,

I did it as below:

3^x can be grouped as a unit as say y.

So the equation now becomes :

y - y/3 = 162.

Solving this gets me y = 243.
So y = 3^x = 243 which solves as to x =5.

x(x-1) = 5 X 4 = 20 (C)

by sanp_l

Tue Aug 31, 2010 11:38 pm
Forum: Problem Solving
Topic: Exponent question
Replies: 10
Views: 1952

This isn't a routine CR question with five answer choices. Otherwise i would surely have put in everything. It is just CR passage. The question is to answer whether the reasoning used by the author of the passage is strong or weak. I hope i have made things clear. I wasn't convinced with the answer ...

by sanp_l

Wed Aug 18, 2010 9:33 am
Forum: Critical Reasoning
Topic: CR argurment: Weak or Strong
Replies: 7
Views: 1609

CR argurment: Weak or Strong

Hi,

Please discuss your takes on whether the argument is strong/weak.

Will be posting the OA after some discussion.



Image



Thanks.

by sanp_l

Tue Aug 17, 2010 6:49 pm
Forum: Critical Reasoning
Topic: CR argurment: Weak or Strong
Replies: 7
Views: 1609

Hi, My understanding is as below: Firstly, The value for x^3(1 - x^2) depends on a) x^3 is +ve/-ve b) x^2 is greater than or less than 1. If x < 0, x^3 < 0. But (1-x^2) depends on the exact value of x. It may be that x < 1 or x < 0 If x > 0, (1- X^2) has to be < 0. Thus x < 1. Hence Insufficient. Se...

by sanp_l

Sun Aug 08, 2010 11:13 pm
Forum: Data Sufficiency
Topic: Is x negative?
Replies: 8
Views: 1624

Received a PM asking me to reply. (FYI for future: we get tons of requests. It's unusual to get a same-day response - we just get too many requests!.) Your test is in 4 days. Your highest practice test score is a 640. You want a 740. As you suspected, that is not going to happen in 4 days. A 100 po...

by sanp_l

Thu Aug 05, 2010 1:10 pm
Forum: GMAT Strategy
Topic: Urgent Expert Advice needed. Please help
Replies: 8
Views: 1263

There are so many knowledgeable souls out there....can some expert help me with my queries ?
Please...

by sanp_l

Thu Aug 05, 2010 8:41 am
Forum: GMAT Strategy
Topic: Urgent Expert Advice needed. Please help
Replies: 8
Views: 1263

@uwhusky: Thanks for your vote. I am surely going to do that.

Can anyone tell me how to proceed from here..? AM desperately looking for some help..

by sanp_l

Mon Aug 02, 2010 8:44 am
Forum: GMAT Strategy
Topic: Urgent Expert Advice needed. Please help
Replies: 8
Views: 1263

Urgent Expert Advice needed. Please help

Hi All, My GMAT is scheduled on 9th AUG and I need a 740+. After one and half months of study the following are my scores from 5 Manhattan Tests which i have given till date: MGMAT1: 590 (Wasn't able to complete the exam in one sitting, was tired, this is the free test available from manhattan) MGMA...

by sanp_l

Sun Aug 01, 2010 11:36 pm
Forum: GMAT Strategy
Topic: Urgent Expert Advice needed. Please help
Replies: 8
Views: 1263

Regarding Quant Practice

Hi All, Can you suggest me any books for Quant. My concern is i am done with OG. The Quant review from GMAC would also have the majority of the questions from the OG itself. And while taking Manhattan Tests, i realized that i have not grilled myself with much of the difficult stuff. When i say diffi...

by sanp_l

Wed Jul 14, 2010 7:05 pm
Forum: GMAT Math
Topic: Regarding Quant Practice
Replies: 0
Views: 1069

Hi, Option 1 is sufficient to answer the question. pv< 0. hence, either p <0 or v < 0. It is given that p > m. So pv > mv can only be true when v has a +ve sign. If v is -ve, then p < m. hence, because p > m is given and that pv >mv, v > 0. From this it follows that p < 0 which is stated in Option 2...

by sanp_l

Fri Jun 25, 2010 8:51 am
Forum: Data Sufficiency
Topic: DS.... GMATPrep Prac Test 2
Replies: 2
Views: 1088

I share your concern. LSAT CR tests are tougher and 20 questions in 30 minutes seems to be an unachievable feat.
Experts please comment...

by sanp_l

Mon Jun 21, 2010 11:00 am
Forum: Critical Reasoning
Topic: Question about LR bible and LSAT preptests
Replies: 2
Views: 1147

I would go with Option D. The first is a claim in support of the argument; the second is a piece of evidence against the argument. The scientists attribute the low rate of cancer to intake of Omega acids where as the second piece states that the concerned fish e.g. the lean fish is rarely rich in fa...

by sanp_l

Sat Jun 12, 2010 10:25 am
Forum: Critical Reasoning
Topic: bold face
Replies: 9
Views: 3850